LSAT and Law School Admissions Forum

Get expert LSAT preparation and law school admissions advice from PowerScore Test Preparation.

 SherryZ
  • Posts: 124
  • Joined: Oct 06, 2013
|
#12709
Hi guys, thank you so much for your help! Let's nail the Dec LSAT together!

Dec 2000 LSAT, Sec 3 LR, Q21:

I eliminated answer choices A, B, D, E, and confidently chose C! (Well...ironically, the correct answer choice is A).

So I reread this question but I still don't get why A is right. Then I tried to use NEGATE technic to figure out why C is wrong but A is right, unluckily, I still don't get it :(

Could you analyze this question and explain why A is right but C is wrong?

Thanks again!

---Sherry
 BethRibet
PowerScore Staff
  • PowerScore Staff
  • Posts: 200
  • Joined: Oct 17, 2012
|
#12794
Hi Sherry,

You're welcome!

Answer choice C is a general statement about what is usually true; we don't have any information that tells us that it certainly applies to this case. Therefore, even if C is true, we do not know whether the conclusion of the stimulus is certain/justified.

Answer choice A speaks directly to the conditional relationship in the last sentence. That is:

If nations are to prosper (sufficient) then economists must look beyond their own borders. (necessary)

or

NP --> ELBB

The first sentence also indicates that the question of "influence" on economies bears on the question of how economists view their home economies. So the assumption that follows here is that for a nation to prosper, economists must understand all influences, not just domestic ones. If answer choice A is taken as true, then the conclusion, that prosperity is dependent on what economists understand, follows logically.

Hope this explains!

Beth
 SherryZ
  • Posts: 124
  • Joined: Oct 06, 2013
|
#12801
Dear Beth,

Thank you again for your reply!

I chose C because my logic was: Because the economic theories depend on idealizations are less accurate :arrow: So gov economists should look beyond borders "

Could you tell me what error I made above? Thank you so much!

---Sherry
User avatar
 KelseyWoods
PowerScore Staff
  • PowerScore Staff
  • Posts: 1079
  • Joined: Jun 26, 2013
|
#12865
Hi Sherry!

The conditional statement in this stimulus is the conclusion of the argument (the second half of the last sentence), as Beth already diagrammed. So one error with the logic you've diagrammed is that it doesn't represent the conditional statement given in the stimulus. Remember to use those sufficient and necessary indicator words when they are given. "If" is a common sufficient indicator and "must" is a common necessary indicator.

Once you identify that conclusion and see that it is conditional, you should focus on it. There's something mentioned in the conclusion that is "new" information: the idea of "to prosper." The key here is not about whether or not the theories are accurate. It's about whether they can help their nations prosper. That's the key component missing from your logic: why should they look beyond the borders? It's not just because theories based on idealizations are less accurate. Looking beyond borders is necessary for a nation to prosper.

Another thing to keep in mind: this is a Justify question. The stem says "follows logically IF assumed." When you have "if assumed," it isn't an assumption question and "the conclusion follows logically" is a common phrase in Justify question stems. Since this is a Justify question and not an Assumption question, you can't use the Assumption Negation technique (it only works for Assumption questions!).

So we should be using the Justify Formula:
Premises + Answer Choice = Conclusion

Premises: EESI :arrow: ELBB
(this is the contrapositive of what the premises basically say, that if economists aren't looking beyond borders, then they aren't taking into account every significant influence)
Answer Choice (A): NP :arrow: EESI
(if nation is prospering, they must examine every significant influence)

If you link the Premises and Answer Choice (A) together, you get:
NP :arrow: EESI :arrow: ELBB

Which brings you to....

Conclusion: NP :arrow: ELBB
(if nation is prospering, economists must look beyond borders)

So the addition of answer choice (A) proves our conclusion. Answer choice (C) is not nearly strong enough to prove that for a nation to prosper, the government economists need to look beyond their borders.

Hope that helps!

Best,
Kelsey
 Jiya
  • Posts: 15
  • Joined: Aug 15, 2014
|
#16990
Hi there,

I selected E as the correct answer because I thought the use of "every" in answer A was a little too extreme. Can you please help me understand why A is the right answer and not E?

Thanks!
 David Boyle
PowerScore Staff
  • PowerScore Staff
  • Posts: 836
  • Joined: Jun 07, 2013
|
#16994
Jiya wrote:Hi there,

I selected E as the correct answer because I thought the use of "every" in answer A was a little too extreme. Can you please help me understand why A is the right answer and not E?

Thanks!
Hello Jiya,

Answer E is tempting, but even if some (say, 2!) economists out of the whole government happen to ignore international trade, is that really going to affect things a lot? much less justify the stimulus?
Answer A, though, connects things as needed. The stimulus:

Premise: international trade important ("significant influence")
Conclusion: prosper :arrow: economists must look beyond national borders (e.g., international trade)

So, answer A, prosper :arrow: economists must look at every influence, makes the needed connection, since otherwise, maybe they could miss one or two influences and the economy could still prosper.

Hope this helps,
David
 sodomojo
  • Posts: 24
  • Joined: Aug 01, 2017
|
#41199
KelseyWoods wrote:Another thing to keep in mind: this is a Justify question. The stem says "follows logically IF assumed." When you have "if assumed," it isn't an assumption question and "the conclusion follows logically" is a common phrase in Justify question stems. Since this is a Justify question and not an Assumption question, you can't use the Assumption Negation technique (it only works for Assumption questions!).

So we should be using the Justify Formula:
Premises + Answer Choice = Conclusion
Sorry I'm not following... why is this not a SA question?

Doesn't "Premises + AC = Conclusion" mean that the correct AC is sufficient to allow the argument to conclude as it does?
 Eric Ockert
PowerScore Staff
  • PowerScore Staff
  • Posts: 164
  • Joined: Sep 28, 2011
|
#41760
Hi sodomojo!

When you say "SA question", I think you are referring to a "Sufficient Assumption" question. If so, this isn't a term that we use in our course materials. You may be familiar with this phrasing from some other materials.

That being said, what you would refer to as Sufficient Assumption question is the same thing as what we would refer to as a Justify the Conclusion question. So the names might be different, but the ideas are the same.

Hope that clears things up a bit!
 sodomojo
  • Posts: 24
  • Joined: Aug 01, 2017
|
#41774
Eric Ockert wrote:Hi sodomojo!

When you say "SA question", I think you are referring to a "Sufficient Assumption" question. If so, this isn't a term that we use in our course materials. You may be familiar with this phrasing from some other materials.

That being said, what you would refer to as Sufficient Assumption question is the same thing as what we would refer to as a Justify the Conclusion question. So the names might be different, but the ideas are the same.

Hope that clears things up a bit!
Ah gotcha! The LR Bible was the first thing I hit during my prep process. It's been a while and I've used other materials to supplement since then, so my memory on PS terms are a little hazy. Thanks for clearing that up.
 bonnie_a
  • Posts: 32
  • Joined: Jun 05, 2021
|
#90173
Hello, I have a question about the reasoning structure of this argument. When there's an argument with a conclusion that goes "just as A is, B should do so if they are to do something," what's the role of the first part ("just as A is,")? I somehow thought this could be a supporting premise for the conclusion but I noticed other premises presented earlier in the stimulus are the ones that support the conclusion. In short, my question is: what is the role of the first half of the main conclusion here? Thank you so much as always!

Get the most out of your LSAT Prep Plus subscription.

Analyze and track your performance with our Testing and Analytics Package.